آيا شما مايل به راه افتادن تاپيك ماراتن نامساوي مي باشي


  • مجموع رای دهندگان
    339

Dadgarnia

New Member
ارسال ها
1,350
لایک ها
1,127
امتیاز
0
پاسخ : ماراتن نامساوي

سوال بعد:
a,b,c سه تا عدد بزرگتر از صفر هستن و داریم
ثابت کنید:

ابتدا عبارت بالا رو به صورت خطي در مياريم:
حالا قرار مي ديم
پس داريم:

پس با استفاده از
كافيه نامساوي رو به ازاي
ثابت كنيم كه در اين حالت درستي نامساوي بديهي است.​

---- دو نوشته به هم متصل شده است ----

سوال بعد:
براي اعداد حقيقي و مثبت
نشان دهيد:

 

m-saghaei

New Member
ارسال ها
338
لایک ها
258
امتیاز
0
پاسخ : ماراتن نامساوي

سوال بعد:
براي اعداد حقيقي و مثبت
نشان دهيد:

اول بازش میکنیم:


پس:


حالا چون روابط روبه رو رو داریم:


نتیجه میشه که:



حالا میخوایم بدونیم آیا رابطه زیر برقراره؟



که اینم اگه ساده اش کنیم میشه آیا داریم
؟
که اینم بدیهیه.چون هر کدوم طبق حسابی هندسی بزرگتر مساوی 3 هستن.
 

Dadgarnia

New Member
ارسال ها
1,350
لایک ها
1,127
امتیاز
0
پاسخ : ماراتن نامساوي

اول بازش میکنیم:


پس:


حالا چون روابط روبه رو رو داریم:


نتیجه میشه که:



حالا میخوایم بدونیم آیا رابطه زیر برقراره؟



که اینم اگه ساده اش کنیم میشه آیا داریم
؟
که اینم بدیهیه.چون هر کدوم طبق حسابی هندسی بزرگتر مساوی 3 هستن.
يه راه ديگش هم اينه كه كلا بازش كنيم:
حالا با استفاده از مورهد داريم
و با استفاده از حسابي - هندسي
كه حكم رو نتيجه ميده. كلا از اين جور كارا خوشم مياد :4:.
 
آخرین ویرایش توسط مدیر

m-saghaei

New Member
ارسال ها
338
لایک ها
258
امتیاز
0
پاسخ : ماراتن نامساوي

سوال بعد:
اگر
سه عدد حقیقی نامنفی باشن و داشته باشیم
ثابت کنید:

 

TheOverlord

New Member
ارسال ها
159
لایک ها
282
امتیاز
0
پاسخ : ماراتن نامساوي

يه راه ديگش هم اينه كه كلا بازش كنيم:
حالا با استفاده از مورهد داريم
و با استفاده از حسابي - هندسي
كه حكم رو نتيجه ميده. كلا از اين جور كارا خوشم مياد :4:.
تو استفاده از مورهد دقت كنيد چرا كه نامساويتون متقارن نيست و مورهد متقارنه.
 

Dadgarnia

New Member
ارسال ها
1,350
لایک ها
1,127
امتیاز
0
پاسخ : ماراتن نامساوي

تو استفاده از مورهد دقت كنيد چرا كه نامساويتون متقارن نيست و مورهد متقارنه.
نگاه كنيد مگه تعريف مورهد اين نيست:
واضحه كه ميشه
ها رو جوري تعيين كرد كه نامساوي متقارن نباشه.
 

TheOverlord

New Member
ارسال ها
159
لایک ها
282
امتیاز
0
پاسخ : ماراتن نامساوي

دقت کنید که سیگمای مذکور، سیگمای جایگشتی است، نه دوری یعنی روی همه جایگشت ها محاسبه میگردد.
 

sepidfekr

New Member
ارسال ها
711
لایک ها
637
امتیاز
0
پاسخ : ماراتن نامساوي

سوال بعد:
 

Dadgarnia

New Member
ارسال ها
1,350
لایک ها
1,127
امتیاز
0
پاسخ : ماراتن نامساوي

يه راهي پيدا كردم خودم خيلي حال كردم چون خيلي وقت بود با نامساوي هاي كلاسيك هـيچي حل نكرده بودم:
با استفاده از نامساوي حسابي - توافقي ( اين نامساوي رو ميگم
اسمش همينه؟) داريم:
پس براي اثبات حكم كافيه ثابت كنيم:
با استفاده از شرط سوال ميدونيم
و همچنين با استفاده از حسابي - توافقي (همون نامساوي بالا) داريم
و با استفاده از كوشي بدست مياد:
پس حكم ثابت ميشه.


 
آخرین ویرایش توسط مدیر

Dadgarnia

New Member
ارسال ها
1,350
لایک ها
1,127
امتیاز
0
پاسخ : ماراتن نامساوي

سوال بعد:
اگر
سه عدد حقیقی نامنفی باشن و داشته باشیم
ثابت کنید:

سوال خيلي سختي بود:18:!
فرض مي كنيم
با استفاده از حسابي - هندسي داريم:
از طرف ديگه با حسابي - هندسي داريم:


پس بايد ثابت كنيم:
بازم با حسابي - هندسي داريم:
پس حكم ثابت ميشه.




 
آخرین ویرایش توسط مدیر

math1998

New Member
ارسال ها
336
لایک ها
224
امتیاز
0
پاسخ : ماراتن نامساوي

سوال خيلي سختي بود:18:!
فرض مي كنيم
با استفاده از حسابي - هندسي داريم:
از طرف ديگه با حسابي - هندسي داريم:

پس بايد ثابت كنيم:
بازم با حسابي - هندسي داريم:
پس حكم ثابت ميشه.




نامساوی اکید شده!!!!

---- دو نوشته به هم متصل شده است ----

سوال خيلي سختي بود:18:!
فرض مي كنيم
با استفاده از حسابي - هندسي داريم:
از طرف ديگه با حسابي - هندسي داريم:

پس بايد ثابت كنيم:
بازم با حسابي - هندسي داريم:
پس حكم ثابت ميشه.




نامساوی اکید شده!!!!
 

Dadgarnia

New Member
ارسال ها
1,350
لایک ها
1,127
امتیاز
0
پاسخ : ماراتن نامساوي

نامساوی اکید شده!!!!

---- دو نوشته به هم متصل شده است ----



نامساوی اکید شده!!!!
به نظر شما اشكالي توي راه من وجود داره؟ اگه راه من اشكال نداشته باشه خب نامساوي اكيد بشه مگه مشكلي ايجاد مي كنه؟ ولي نامساوي هاي قويتري رو هم ميشه با همين شروط سوال ثابت كرد مثل اين:
كه شايد اينا اكيد نباشن و حالت تساوي وجود داشته باشه.
 

Dadgarnia

New Member
ارسال ها
1,350
لایک ها
1,127
امتیاز
0
پاسخ : ماراتن نامساوي

سوال بعد:
اعداد مثبت
در رابطه ی
صدق می کنند. حداکثر عبارت زیر را بیابید:

 

REZA 73

Active Member
ارسال ها
139
لایک ها
184
امتیاز
43
پاسخ : ماراتن نامساوي

سوال بعد:
اعداد مثبت
در رابطه ی
صدق می کنند. حداکثر عبارت زیر را بیابید:

کافیه پرانتز ها رو در هم ضرب کنیم
مقدار ماکزیمم 1/8 است پس باید ثابت کنیم:





در نتیجه سوال اثبات شده است.

---- دو نوشته به هم متصل شده است ----
 
آخرین ویرایش توسط مدیر

Dadgarnia

New Member
ارسال ها
1,350
لایک ها
1,127
امتیاز
0
پاسخ : ماراتن نامساوي

کافیه پرانتز ها رو در هم ضرب کنیم
مقدار ماکزیمم 1/8 است پس باید ثابت کنیم:





در نتیجه سوال اثبات شده است.

---- دو نوشته به هم متصل شده است ----
میشه در مورد نامساوی های خط دوم و سوم بیشتر توضیح بدین؟ در ضمن شما حالت تساوی رو هم معین نکردین.
 

REZA 73

Active Member
ارسال ها
139
لایک ها
184
امتیاز
43
پاسخ : ماراتن نامساوي

میشه در مورد نامساوی های خط دوم و سوم بیشتر توضیح بدین؟ در ضمن شما حالت تساوی رو هم معین نکردین.
اولا که فرض کردیم a بزرگتر مساویه b. و b بزرگتر مساویه c پس خط دوم ثابت میشه خط سوم هم که یه. یه حسابی هندسی ساده است حالت تساوی هم برای 1و 0.5 و 0.5 به دست می آید،اگه دقت کنید حالت تساوی ها همه بر روی نامساوی ها رعایت شده است.
 

Dadgarnia

New Member
ارسال ها
1,350
لایک ها
1,127
امتیاز
0
پاسخ : ماراتن نامساوي

اولا که فرض کردیم a بزرگتر مساویه b. و b بزرگتر مساویه c پس خط دوم ثابت میشه خط سوم هم که یه. یه حسابی هندسی ساده است حالت تساوی هم برای 1و 0.5 و 0.5 به دست می آید،اگه دقت کنید حالت تساوی ها همه بر روی نامساوی ها رعایت شده است.
نامساوی خط دوم در صورتی درسته که a بزرگتر از یک باشه و نامساوی خط سوم ضریب abc میشه 4 نه 16 همچنین شما از شرط سوال چه استفاده ای کردین؟
 
آخرین ویرایش توسط مدیر

REZA 73

Active Member
ارسال ها
139
لایک ها
184
امتیاز
43
پاسخ : ماراتن نامساوي

نامساوی خط دوم در صورتی درسته که a بزرگتر از یک باشه و نامساوی خط سوم ضریب abc میشه 4 نه 16 همچنین شما از شرط سوال چه استفاده ای کردین؟
از شرط مساله که برای به دست آوردن
استفاده شده
فقط کافیه با استفاده دوباره از شرط مساله ثابت بشه که
که در پست اول اشتباه نوشتمb+c
 

Dadgarnia

New Member
ارسال ها
1,350
لایک ها
1,127
امتیاز
0
پاسخ : ماراتن نامساوي

از شرط مساله که برای به دست آوردن
استفاده شده
فقط کافیه با استفاده دوباره از شرط مساله ثابت بشه که
که در پست اول اشتباه نوشتمb+c
بله حق با شماست نمی دونم چرا امشب اینقدر خنگ شدم :4: ولی این نامساوی آخر رو توضیح ندادین من فکر می کنم شما فرض کردین b بزرگتر مساوی یکه.
 

REZA 73

Active Member
ارسال ها
139
لایک ها
184
امتیاز
43
پاسخ : ماراتن نامساوي

بله حق با شماست نمی دونم چرا امشب اینقدر خنگ شدم :4: ولی این نامساوی آخر رو توضیح ندادین من فکر می کنم شما فرض کردین b بزرگتر مساوی یکه.
من فرض خاصی نکردم ولی چیزی که مشهوده اگه a بزرگتر از یک باشه اون نامساوی درسته، البته میشه با روش لاگرانژ درستی این نابرابری جزئی رو به راحتی چک کرد، من با لاگرانژ بررسی کردم، ولی به احتمال زیاد یه اثبات خیلی ساده تر داره، متاسفانه به دلیل درس های زیاد دانشگاه فرصت نمیشه بیشتر از این رو مسایل فک کنم .
 
بالا